Đến nội dung

ZzNightWalkerZz

ZzNightWalkerZz

Đăng ký: 20-04-2015
Offline Đăng nhập: 13-07-2018 - 16:06
***--

#585732 CM: $\forall n\in mathbb{N}$ thì $B= 9n^3...

Gửi bởi ZzNightWalkerZz trong 29-08-2015 - 15:23

Đặt $P(n)=n+1)(n+2)(n+3)......(n+n)$
Ta có $P(1)=2\vdots 2^1$ (đúng).Giả sử $P(n)\vdots 2^n$ với mọi $n$ là số tự nhiên
Lại có: $P(n+1)=P(n).2\vdots 2^n.2=2^{n+1}$
Vậy theo nguyên lí qui nạp ta có đpcm

Ở dòng thứ 3 phải là : $P(n+1)=P(n).2(2n+1)$ 




#583661 $3\left [ x^{2} \right ]+5\left [ x \right...

Gửi bởi ZzNightWalkerZz trong 21-08-2015 - 17:42

Thật sự là phần này đúng ạ. Khi nào nào vậy mới sai nhá: $x^{2}=([x])^{2}$.

 

$\left [ x^{2} \right ]=\left [ x \right ].\left [ x \right ]$ ?

P/s: Mình đang phân vân đoạn này

Chỗ này thực sự chưa đúng : với $x=3,9$ thì $[x^2]=15, [x]^2=9$

Bài trên mình có cách giải riêng (Chưa xem tài liệu)

Ta luôn có : $x^2\geq [x]^2$ mà $[x^2]$ là số nguyên dương lớn nhất không quá $x^2$ nên $[x^2]\geq [x]^2$

$=> 2\geq 3[x]^2+5[x]<=>0\geq (3[x]-1)([x]+2)=>\frac{1}{3}\geq [x]\geq -2$

Đến đây coi như xong




#582728 Tìm các số tự nhiên $a,b,c,d$ thoả mãn $ 4^a.5^b - 3^c.11^d =...

Gửi bởi ZzNightWalkerZz trong 17-08-2015 - 22:18

Tìm các số tự nhiên $a,b,c,d$ thoả mãn $ 4^a.5^b - 3^c.11^d = 1.$

Một nghiệm của phương trình là $a=c=1, b=d=0$

Các trường hợp $a=0$ với $b=0$ có lẽ không cần nói đến

Với $d=0$

Nếu $c=0$ thì vô lí $=>c\neq 0=> c$ lẻ (ai đọc tự hiểu :D) $=> 3^c+1\not \vdots 5$ 

Xét với $d>0$ thì chỉ cần bổ đề sau là được 

Bổ đề

Giải nốt sẽ hơi nhàm  :icon6:

Lời giải còn sơ sài, thiếu chỗ nào mọi người thông cảm  :mellow:




#582699 TÌm GTLN,GTNN của S=(2-x)(2-y)

Gửi bởi ZzNightWalkerZz trong 17-08-2015 - 21:10

Mình mới tìm ra min thôi:

Ta có:

$2S= 8+2xy-4(x+y)\Leftrightarrow 2S+1= (x+y)^{2}-4(x+y)+8\geq 4$

$\Leftrightarrow S\geq \frac{3}{2}.$ :closedeyes:

Xem lại dấu "="

Biến đổi như trên thì ta chỉ cần phải tìm min, max của : $(x+y)^2-4(x+y)+4=(2-x-y)^2$

Dễ dàng nhận thấy $2>x+y$ nên ta chỉ phải tìm min, max của $2-x-y>0$

Mà $(x+y)^2=1+2xy\leq 1+\frac{(x+y)^2}{2}<=>(x+y)^2\leq 2<=> -\sqrt{2}\leq x+y\leq \sqrt{2}$

Vậy $S_{min}$ khi $x=y=\frac{\sqrt{2}}{2}$, $S_{max}$ khi $x=y=-\frac{\sqrt{2}}{2}$




#577601 Tính $M=x-y$

Gửi bởi ZzNightWalkerZz trong 01-08-2015 - 21:42

Dễ mờ, làm gì kinh khủng thế.

Cộng, trừ lần lượt hai đẳng thức ta có :

$\frac{2}{y}=y^2+3x^2;\frac{10}{x}=3y^2+x^2<=>10=3y^2x+x^3;2=3x^2y+y^3$ 

Trừ hai vế : $8=x^3-3x^2y+3y^2-y^3=(x-y)^3<=>x-y=2$




#577562 $\frac{1}{x_{1}}+\frac{2}{x_{1}+x_{2}}+...+\frac{n}{...

Gửi bởi ZzNightWalkerZz trong 01-08-2015 - 20:20

Đề ban đầu sai thì phải, cho $x_1\geq x_2...\geq x_n$ thì bài toán đổi chiều




#576917 Chứng minh rằng $n$ có không ít hơn một ước số nguyên tố thì ...

Gửi bởi ZzNightWalkerZz trong 30-07-2015 - 21:41

Đề bài chưa đúng cho lắm, phải là $n$ có trên 1 ước số nguyên tố mới đúng

$\boxed{\text{Chiều thuận}}$

Do $n$ có trên 1 ước số nguyên tố nên luôn phân tích $n$ thành tích $p.q$ với $(p,q)=1$

Do $(k,k+1)=1$ nên ta phải chứng minh tồn tại số nguyên $k$ thỏa mãn hai điều kiện

$$k=pk_1;k+1=qk_2$$

Giả sử trong dãy số : $q,2q,3q...(p-1)q$ có 2 số có cùng số dư khi chia cho $p$. Giả sử hai số đó là $m_1q, m_2q$ ($1\leq m_1<m_2\leq p-1$)

$=>(m_2-m_1)q\vdots p$ (Vô lí do $(p,q)=1, m_2-m_1<p$)

Vậy trong $p-1$ số trên không có số nào có cùng số dư khi chia cho $p$ nên tồn tại ít nhất một số chia cho $p$ dư 1, ta đã chứng minh được tồn tại số nguyên tố $k$ thỏa mãn.

$\boxed{\text{Chiều nghịch}}$

Giả sử $n$ chỉ có một ước số nguyên tố, khi đó $n$ có dạng $n=p^x$ với $p$ là một số nguyên tố

Do $(k,k+1)=1$ nên chỉ có một số chia hết cho $n$ nên $k\geq n$ hoặc $k+1\geq n$ (Vô lí)

Vậy $n$ có trên 1 ước số nguyên tố

 




#576360 $p.x^2+q.y^2=m.z^2$

Gửi bởi ZzNightWalkerZz trong 28-07-2015 - 22:49

Giải phương trình $p.x^2+q.y^2=m.z^2$ với $x,y,z$ là các số tự nhiên, $p,q,m$ là các số nguyên tố cho trước.

.




#574331 $2^m+1$ không chia hết cho $2^n-1$

Gửi bởi ZzNightWalkerZz trong 20-07-2015 - 21:36

$\texttt{Solution}$

 

$\blacklozenge$ Nếu $m<n$ thì với $n>2$ ta có : $2^m+1<2^n-1$

Suy ra $2^m+1$ không chia hết cho $2^n-1$

 

$\blacklozenge$ Nếu $m=n$ thì $\frac{2^m+1}{2^n-1}=1+\frac{2}{2^n-1}$ 

Với $n>2$ thì  $\frac{2^m+1}{2^n-1}=1+\frac{2}{2^n-1}$  không phải là số nguyên. 

Suy ra $2^m+1$ không chia hết cho $2^n-1$

 

$\blacklozenge$ Nếu $m>n$. Ta đặt $m=kn+h$ ($k,h$ nguyên dương, $r$ tự nhiên và nhỏ hơn $n$.

Khi đó : $\frac{2^m+1}{2^n-1}=\frac{2^m-2^r}{2^n-1}+\frac{2^r+1}{2^n-1}=\frac{2^m-2^{m-kn}}{2^n-1}+\frac{2^r+1}{2^n-1}=\frac{2^{m-kn}(2^{kn}-1)}{2^n-1}+\frac{2^r+1}{2^n-1}$

Nhận xét : $\frac{2^{m-kn}(2^{kn}-1)}{2^n-1}$ nguyên. Vì $r<n$ nên theo chứng minh trên thì $\frac{2^r+1}{2^n-1}$ không nguyên

Suy ra $2^m+1$ không chia hết cho $2^n-1$

 

Vậy trong mọi trường hợp thì $2^m+1$ không chia hết cho $2^n-1$    $\square$

Cách giải rất hay nhưng $h$ biến mất đâu rồi anh?  :icon6:




#572847 $n^2+2^n\vdots 2p$

Gửi bởi ZzNightWalkerZz trong 15-07-2015 - 20:51

$\texttt{Bài toán}$ Cho $p$ là số nguyên tố có dạng $p=4k+1$. Hỏi có tồn tại hay không số tự nhiên $n$ sao cho $n^2+2^n\vdots 2p$

 

Gợi ý




#572208 $\sum_{k=1}^{n^2}\left \{ \...

Gửi bởi ZzNightWalkerZz trong 13-07-2015 - 23:06

$\texttt{Solution}$

Spoiler


Ta sẽ chứng minh khẳng định bằng quy nạp theo $n$.
Với $n=1$ thì hiển nhiên khẳng đinh đúng. Giả sử khẳng định đúng với $n$ , ta chứng minh khẳng định đúng với $n+1$. Ta có nhận xét :
$n<\sqrt{n^2+1}<\sqrt{n^2+2}<...<\sqrt{n^2+2n}<n+1$
nên $\forall i=1,2,..,2n$ ta có : $\left \{\sqrt{n^2+i} \right \} =\sqrt{n^2+i}-n<\sqrt{n^2+i+\frac{i^2}{4n^2}}-n=\frac{i}{2n}$
Ta có :
$\sum_{k=1}^{(n+1)^2}\left \{ \sqrt{k} \right \}=\sum_{k=1}^{n^2}\left \{ k \right \}+\sum_{k=n^2+1}^{(n+1)^2}\left \{ k \right \}$
$< \frac{n^2-1}{2}+\frac{\sum_{i=1}^{2n}i}{2n}$
$= \frac{n^2-1}{2}+\frac{2n+1}{2}=\frac{(n+1)^2-1}{2}$
Suy ra khẳng định đúng với $n+1$
Vậy bài toán được chứng minh $\square$

Chuẩn cách em. Đang định gửi thì a trả lời : D


#571015 $\sum \sqrt{a_i} \notin Q$

Gửi bởi ZzNightWalkerZz trong 10-07-2015 - 12:57

Chứng minh rằng với các số $a_i(i=1,2,...,n)$ không là số chính phương thì 

$\sum \sqrt{a_i} \notin Q$

 

P/s : Nếu thánh nào có khả năng chém chuối bá đạo thì xin mời bài tiếp :D

Với các số $a_i(i=1,2,...,n)$ không có dạng $n^m(m,n\in N)$ thì

$\sum \sqrt[m]{a_i} \notin Q$

 


  • Nxb yêu thích


#570626 $x^2+y^2=2^k z^2$

Gửi bởi ZzNightWalkerZz trong 08-07-2015 - 22:10

Tìm nghiệm nguyên dương của phương trình $x^2+y^2=2^z z^2$

Spoiler




#569011 $\sum \frac{tanA}{tanB}\geq \sum...

Gửi bởi ZzNightWalkerZz trong 29-06-2015 - 23:49

Ta có : $tan^2A+1=\frac{1}{cos^2A}$

BĐT cần chứng minh tương đương với
$\sum \frac{tanA}{tanB}(1-\frac{cos^2A}{cos^2B})\geq0<=>\sum \frac{tanA}{tanB}(tan^2A-tan^2B)(tan^2A+1)\geq0$

Không mất tính tổng quát, giả sử $\widehat A\geq\widehat B\geq\widehat C>0$

Ta sẽ chứng minh hai BĐT

$+) \sum \frac{tanA}{tanB}(tan^2A-tan^2B)\geq0<=>(tan^2A-tan^2B)(\frac{tanA}{tanB}-\frac{tanC}{tanA})+(tan^2B-tan^2C)(\frac{tanB}{tanC}-\frac{tanC}{tanA})\geq0$

Do $tan^2A\geq tanB.tanC$, $tanB.tanA\geq tan^2C$ nên BĐT trên đã rõ

$+) \sum \frac{tan^3A}{tanB}(tan^2A-tan^2B)\geq0<=>(tan^2A-tan^2B)(\frac{tan^3A}{tanB}-\frac{tan^3C}{tanA})+(tan^2B-tan^2C)(\frac{tan^3B}{tanC}-\frac{tan^3C}{tanA})\geq0$

Cũng tương tự như trên bất đẳng thức luôn đúng

Cộng hai bất đẳng thức trên ta có điều phải chứng minh




#568780 cho a, b thỏa mãn : $\left\{\begin{matrix}...

Gửi bởi ZzNightWalkerZz trong 28-06-2015 - 22:15

Biến đổi tương đương thì bất đẳng thức trên tương đương với

$\frac{1-2a}{1-2b}+\frac{1-2b}{1-2a}\leq \frac{1-a}{1-b}+\frac{1-b}{1-a}<=>\frac{a-b}{(1-2b)(1-b)}+\frac{b-a}{(1-2a)(1-a)}\geq0$

Không mất tính tổng quát, giả sử $a\geq b=>2a>1$. Bây giờ ta cần phải chứng minh

$\frac{1}{(1-2a)(1-a)}-\frac{1}{(1-2b)(1-b)}\leq0$

Nếu như $b\leq\frac{1}{2}$ thì điều phải chứng minh đã rõ

Nếu $b>\frac{1}{2}$ thì ta chỉ cần chứng minh $(1-2b)(1-b)\leq(1-a)(1-2a)<=>(b-a)(2b+2a-3)\leq0$ (Điều này luôn đúng với giả thiết)

Vậy ta có điều phải chứng minh